Identifying a group involving transformation of 4 integers












1












$begingroup$


I'm looking at some messy data, and I found by accident that some properties seem preserved under some transformations of a subset of the variables.



After some work I think I found a base set of transformation from which I can build up all the special transformations. I'd like to figure out what group this is, so I can read more about it and hopefully better understand it.



All values are integers, and the last symmetry is parameterized by an integer $t$:
$$
begin{eqnarray}
(a,b,c,d) &rightarrow& (-a,-b,-c,-d) \
(a,b,c,d) &rightarrow& (a,-b,c,-d) \
(a,b,c,d) &rightarrow& (d,c,b,a) \
(a,b,c,d) &rightarrow& (a,b + at,c + 2bt + a t^2,d + 3ct + 3bt^2 + at^3) \
end{eqnarray}
$$



Can someone help me identify what group this is?





EDIT: I made some progress, but can't quite pull it all together.



I rewrote this in linear algebra. And I decided it might be a bit more natural to apply #2 to #3 for a different choice of a basis transformation (for reasons discussed below):
$$
begin{eqnarray}
(a,b,c,d) &rightarrow& (d,-c,b,-a) \
end{eqnarray}
$$

Okay, we can view the transformation as 4x4 matrices, for example the first one:
$$
begin{bmatrix}
-1 & 0 & 0 & 0 \ 0 & -1 & 0 & 0 \ 0 & 0 & -1 & 0 \ 0 & 0 & 0 & -1
end{bmatrix}
begin{bmatrix}
a \ b \ c \ d
end{bmatrix}
=
begin{bmatrix}
-a \ -b \ -c \ -d
end{bmatrix}
$$

And I can now see that the last parameterized symmetry is just the $t=1$ case as a generator
$$
begin{bmatrix}
1 & 0 & 0 & 0 \ t & 1 & 0 & 0 \ t^2 & 2t & 1 & 0 \ t^3 & 3t^2 & 3t & 1
end{bmatrix}
=
begin{bmatrix}
1 & 0 & 0 & 0 \ 1 & 1 & 0 & 0 \ 1 & 2 & 1 & 0 \ 1 & 3 & 3 & 1
end{bmatrix}^t
% {{1, 0, 0, 0},{1,1,0,0},{1,2,1,0},{1,3,3,1}}^n
$$

So the four generators are:
$$
begin{bmatrix}
-1 & 0 & 0 & 0 \ 0 & -1 & 0 & 0 \ 0 & 0 & -1 & 0 \ 0 & 0 & 0 & -1
end{bmatrix}
,
begin{bmatrix}
1 & 0 & 0 & 0 \ 0 & -1 & 0 & 0 \ 0 & 0 & 1 & 0 \ 0 & 0 & 0 & -1
end{bmatrix}
,
begin{bmatrix}
0 & 0 & 0 & 1 \ 0 & 0 & -1 & 0 \ 0 & 1 & 0 & 0 \ -1 & 0 & 0 & 0
end{bmatrix}
,
begin{bmatrix}
1 & 0 & 0 & 0 \ 1 & 1 & 0 & 0 \ 1 & 2 & 1 & 0 \ 1 & 3 & 3 & 1
end{bmatrix}
$$

The first is just the negative identity matrix, and so commutes with the others. Therefore the full group just has this as a product that is easily factored out, so we can focus on the subgroup without it.



When viewed as blocks of 2x2 matrices, all the non-zero blocks have determinant 1 except for the second generator (which has non-zero blocks of determinant -1). So that is somehow a bit "different", and if this turns out to be a known group I'm guessing there is a "special" version of it without that matrix.



So ... a "special" linear group over the integers with two generators ... SL(2,Z) ?



If that is it, I'm really close. But I'm just guessing. I still don't quite see how it all fits together.










share|cite|improve this question











$endgroup$












  • $begingroup$
    Why do you assume this is a group?
    $endgroup$
    – Don Thousand
    Jan 13 at 4:13










  • $begingroup$
    @DonThousand Because I can compose those base transformations to get another valid transformation.
    $endgroup$
    – PineDoors
    Jan 13 at 4:18










  • $begingroup$
    Which properties are preserved?
    $endgroup$
    – zoli
    Jan 13 at 6:38










  • $begingroup$
    Just a few small observations: the orders of the generators are $2,2,4,infty$. Next, the first generator is the square of the third one, so we can throw it out. Next, the second generator acts by inversion on the last two, so the subgroup generated by the last two is normal of index at most $2$, so we can focus on that.
    $endgroup$
    – verret
    Jan 14 at 1:49


















1












$begingroup$


I'm looking at some messy data, and I found by accident that some properties seem preserved under some transformations of a subset of the variables.



After some work I think I found a base set of transformation from which I can build up all the special transformations. I'd like to figure out what group this is, so I can read more about it and hopefully better understand it.



All values are integers, and the last symmetry is parameterized by an integer $t$:
$$
begin{eqnarray}
(a,b,c,d) &rightarrow& (-a,-b,-c,-d) \
(a,b,c,d) &rightarrow& (a,-b,c,-d) \
(a,b,c,d) &rightarrow& (d,c,b,a) \
(a,b,c,d) &rightarrow& (a,b + at,c + 2bt + a t^2,d + 3ct + 3bt^2 + at^3) \
end{eqnarray}
$$



Can someone help me identify what group this is?





EDIT: I made some progress, but can't quite pull it all together.



I rewrote this in linear algebra. And I decided it might be a bit more natural to apply #2 to #3 for a different choice of a basis transformation (for reasons discussed below):
$$
begin{eqnarray}
(a,b,c,d) &rightarrow& (d,-c,b,-a) \
end{eqnarray}
$$

Okay, we can view the transformation as 4x4 matrices, for example the first one:
$$
begin{bmatrix}
-1 & 0 & 0 & 0 \ 0 & -1 & 0 & 0 \ 0 & 0 & -1 & 0 \ 0 & 0 & 0 & -1
end{bmatrix}
begin{bmatrix}
a \ b \ c \ d
end{bmatrix}
=
begin{bmatrix}
-a \ -b \ -c \ -d
end{bmatrix}
$$

And I can now see that the last parameterized symmetry is just the $t=1$ case as a generator
$$
begin{bmatrix}
1 & 0 & 0 & 0 \ t & 1 & 0 & 0 \ t^2 & 2t & 1 & 0 \ t^3 & 3t^2 & 3t & 1
end{bmatrix}
=
begin{bmatrix}
1 & 0 & 0 & 0 \ 1 & 1 & 0 & 0 \ 1 & 2 & 1 & 0 \ 1 & 3 & 3 & 1
end{bmatrix}^t
% {{1, 0, 0, 0},{1,1,0,0},{1,2,1,0},{1,3,3,1}}^n
$$

So the four generators are:
$$
begin{bmatrix}
-1 & 0 & 0 & 0 \ 0 & -1 & 0 & 0 \ 0 & 0 & -1 & 0 \ 0 & 0 & 0 & -1
end{bmatrix}
,
begin{bmatrix}
1 & 0 & 0 & 0 \ 0 & -1 & 0 & 0 \ 0 & 0 & 1 & 0 \ 0 & 0 & 0 & -1
end{bmatrix}
,
begin{bmatrix}
0 & 0 & 0 & 1 \ 0 & 0 & -1 & 0 \ 0 & 1 & 0 & 0 \ -1 & 0 & 0 & 0
end{bmatrix}
,
begin{bmatrix}
1 & 0 & 0 & 0 \ 1 & 1 & 0 & 0 \ 1 & 2 & 1 & 0 \ 1 & 3 & 3 & 1
end{bmatrix}
$$

The first is just the negative identity matrix, and so commutes with the others. Therefore the full group just has this as a product that is easily factored out, so we can focus on the subgroup without it.



When viewed as blocks of 2x2 matrices, all the non-zero blocks have determinant 1 except for the second generator (which has non-zero blocks of determinant -1). So that is somehow a bit "different", and if this turns out to be a known group I'm guessing there is a "special" version of it without that matrix.



So ... a "special" linear group over the integers with two generators ... SL(2,Z) ?



If that is it, I'm really close. But I'm just guessing. I still don't quite see how it all fits together.










share|cite|improve this question











$endgroup$












  • $begingroup$
    Why do you assume this is a group?
    $endgroup$
    – Don Thousand
    Jan 13 at 4:13










  • $begingroup$
    @DonThousand Because I can compose those base transformations to get another valid transformation.
    $endgroup$
    – PineDoors
    Jan 13 at 4:18










  • $begingroup$
    Which properties are preserved?
    $endgroup$
    – zoli
    Jan 13 at 6:38










  • $begingroup$
    Just a few small observations: the orders of the generators are $2,2,4,infty$. Next, the first generator is the square of the third one, so we can throw it out. Next, the second generator acts by inversion on the last two, so the subgroup generated by the last two is normal of index at most $2$, so we can focus on that.
    $endgroup$
    – verret
    Jan 14 at 1:49
















1












1








1





$begingroup$


I'm looking at some messy data, and I found by accident that some properties seem preserved under some transformations of a subset of the variables.



After some work I think I found a base set of transformation from which I can build up all the special transformations. I'd like to figure out what group this is, so I can read more about it and hopefully better understand it.



All values are integers, and the last symmetry is parameterized by an integer $t$:
$$
begin{eqnarray}
(a,b,c,d) &rightarrow& (-a,-b,-c,-d) \
(a,b,c,d) &rightarrow& (a,-b,c,-d) \
(a,b,c,d) &rightarrow& (d,c,b,a) \
(a,b,c,d) &rightarrow& (a,b + at,c + 2bt + a t^2,d + 3ct + 3bt^2 + at^3) \
end{eqnarray}
$$



Can someone help me identify what group this is?





EDIT: I made some progress, but can't quite pull it all together.



I rewrote this in linear algebra. And I decided it might be a bit more natural to apply #2 to #3 for a different choice of a basis transformation (for reasons discussed below):
$$
begin{eqnarray}
(a,b,c,d) &rightarrow& (d,-c,b,-a) \
end{eqnarray}
$$

Okay, we can view the transformation as 4x4 matrices, for example the first one:
$$
begin{bmatrix}
-1 & 0 & 0 & 0 \ 0 & -1 & 0 & 0 \ 0 & 0 & -1 & 0 \ 0 & 0 & 0 & -1
end{bmatrix}
begin{bmatrix}
a \ b \ c \ d
end{bmatrix}
=
begin{bmatrix}
-a \ -b \ -c \ -d
end{bmatrix}
$$

And I can now see that the last parameterized symmetry is just the $t=1$ case as a generator
$$
begin{bmatrix}
1 & 0 & 0 & 0 \ t & 1 & 0 & 0 \ t^2 & 2t & 1 & 0 \ t^3 & 3t^2 & 3t & 1
end{bmatrix}
=
begin{bmatrix}
1 & 0 & 0 & 0 \ 1 & 1 & 0 & 0 \ 1 & 2 & 1 & 0 \ 1 & 3 & 3 & 1
end{bmatrix}^t
% {{1, 0, 0, 0},{1,1,0,0},{1,2,1,0},{1,3,3,1}}^n
$$

So the four generators are:
$$
begin{bmatrix}
-1 & 0 & 0 & 0 \ 0 & -1 & 0 & 0 \ 0 & 0 & -1 & 0 \ 0 & 0 & 0 & -1
end{bmatrix}
,
begin{bmatrix}
1 & 0 & 0 & 0 \ 0 & -1 & 0 & 0 \ 0 & 0 & 1 & 0 \ 0 & 0 & 0 & -1
end{bmatrix}
,
begin{bmatrix}
0 & 0 & 0 & 1 \ 0 & 0 & -1 & 0 \ 0 & 1 & 0 & 0 \ -1 & 0 & 0 & 0
end{bmatrix}
,
begin{bmatrix}
1 & 0 & 0 & 0 \ 1 & 1 & 0 & 0 \ 1 & 2 & 1 & 0 \ 1 & 3 & 3 & 1
end{bmatrix}
$$

The first is just the negative identity matrix, and so commutes with the others. Therefore the full group just has this as a product that is easily factored out, so we can focus on the subgroup without it.



When viewed as blocks of 2x2 matrices, all the non-zero blocks have determinant 1 except for the second generator (which has non-zero blocks of determinant -1). So that is somehow a bit "different", and if this turns out to be a known group I'm guessing there is a "special" version of it without that matrix.



So ... a "special" linear group over the integers with two generators ... SL(2,Z) ?



If that is it, I'm really close. But I'm just guessing. I still don't quite see how it all fits together.










share|cite|improve this question











$endgroup$




I'm looking at some messy data, and I found by accident that some properties seem preserved under some transformations of a subset of the variables.



After some work I think I found a base set of transformation from which I can build up all the special transformations. I'd like to figure out what group this is, so I can read more about it and hopefully better understand it.



All values are integers, and the last symmetry is parameterized by an integer $t$:
$$
begin{eqnarray}
(a,b,c,d) &rightarrow& (-a,-b,-c,-d) \
(a,b,c,d) &rightarrow& (a,-b,c,-d) \
(a,b,c,d) &rightarrow& (d,c,b,a) \
(a,b,c,d) &rightarrow& (a,b + at,c + 2bt + a t^2,d + 3ct + 3bt^2 + at^3) \
end{eqnarray}
$$



Can someone help me identify what group this is?





EDIT: I made some progress, but can't quite pull it all together.



I rewrote this in linear algebra. And I decided it might be a bit more natural to apply #2 to #3 for a different choice of a basis transformation (for reasons discussed below):
$$
begin{eqnarray}
(a,b,c,d) &rightarrow& (d,-c,b,-a) \
end{eqnarray}
$$

Okay, we can view the transformation as 4x4 matrices, for example the first one:
$$
begin{bmatrix}
-1 & 0 & 0 & 0 \ 0 & -1 & 0 & 0 \ 0 & 0 & -1 & 0 \ 0 & 0 & 0 & -1
end{bmatrix}
begin{bmatrix}
a \ b \ c \ d
end{bmatrix}
=
begin{bmatrix}
-a \ -b \ -c \ -d
end{bmatrix}
$$

And I can now see that the last parameterized symmetry is just the $t=1$ case as a generator
$$
begin{bmatrix}
1 & 0 & 0 & 0 \ t & 1 & 0 & 0 \ t^2 & 2t & 1 & 0 \ t^3 & 3t^2 & 3t & 1
end{bmatrix}
=
begin{bmatrix}
1 & 0 & 0 & 0 \ 1 & 1 & 0 & 0 \ 1 & 2 & 1 & 0 \ 1 & 3 & 3 & 1
end{bmatrix}^t
% {{1, 0, 0, 0},{1,1,0,0},{1,2,1,0},{1,3,3,1}}^n
$$

So the four generators are:
$$
begin{bmatrix}
-1 & 0 & 0 & 0 \ 0 & -1 & 0 & 0 \ 0 & 0 & -1 & 0 \ 0 & 0 & 0 & -1
end{bmatrix}
,
begin{bmatrix}
1 & 0 & 0 & 0 \ 0 & -1 & 0 & 0 \ 0 & 0 & 1 & 0 \ 0 & 0 & 0 & -1
end{bmatrix}
,
begin{bmatrix}
0 & 0 & 0 & 1 \ 0 & 0 & -1 & 0 \ 0 & 1 & 0 & 0 \ -1 & 0 & 0 & 0
end{bmatrix}
,
begin{bmatrix}
1 & 0 & 0 & 0 \ 1 & 1 & 0 & 0 \ 1 & 2 & 1 & 0 \ 1 & 3 & 3 & 1
end{bmatrix}
$$

The first is just the negative identity matrix, and so commutes with the others. Therefore the full group just has this as a product that is easily factored out, so we can focus on the subgroup without it.



When viewed as blocks of 2x2 matrices, all the non-zero blocks have determinant 1 except for the second generator (which has non-zero blocks of determinant -1). So that is somehow a bit "different", and if this turns out to be a known group I'm guessing there is a "special" version of it without that matrix.



So ... a "special" linear group over the integers with two generators ... SL(2,Z) ?



If that is it, I'm really close. But I'm just guessing. I still don't quite see how it all fits together.







group-theory integers combinatorial-group-theory






share|cite|improve this question















share|cite|improve this question













share|cite|improve this question




share|cite|improve this question








edited Jan 13 at 6:01







PineDoors

















asked Jan 13 at 4:01









PineDoorsPineDoors

335




335












  • $begingroup$
    Why do you assume this is a group?
    $endgroup$
    – Don Thousand
    Jan 13 at 4:13










  • $begingroup$
    @DonThousand Because I can compose those base transformations to get another valid transformation.
    $endgroup$
    – PineDoors
    Jan 13 at 4:18










  • $begingroup$
    Which properties are preserved?
    $endgroup$
    – zoli
    Jan 13 at 6:38










  • $begingroup$
    Just a few small observations: the orders of the generators are $2,2,4,infty$. Next, the first generator is the square of the third one, so we can throw it out. Next, the second generator acts by inversion on the last two, so the subgroup generated by the last two is normal of index at most $2$, so we can focus on that.
    $endgroup$
    – verret
    Jan 14 at 1:49




















  • $begingroup$
    Why do you assume this is a group?
    $endgroup$
    – Don Thousand
    Jan 13 at 4:13










  • $begingroup$
    @DonThousand Because I can compose those base transformations to get another valid transformation.
    $endgroup$
    – PineDoors
    Jan 13 at 4:18










  • $begingroup$
    Which properties are preserved?
    $endgroup$
    – zoli
    Jan 13 at 6:38










  • $begingroup$
    Just a few small observations: the orders of the generators are $2,2,4,infty$. Next, the first generator is the square of the third one, so we can throw it out. Next, the second generator acts by inversion on the last two, so the subgroup generated by the last two is normal of index at most $2$, so we can focus on that.
    $endgroup$
    – verret
    Jan 14 at 1:49


















$begingroup$
Why do you assume this is a group?
$endgroup$
– Don Thousand
Jan 13 at 4:13




$begingroup$
Why do you assume this is a group?
$endgroup$
– Don Thousand
Jan 13 at 4:13












$begingroup$
@DonThousand Because I can compose those base transformations to get another valid transformation.
$endgroup$
– PineDoors
Jan 13 at 4:18




$begingroup$
@DonThousand Because I can compose those base transformations to get another valid transformation.
$endgroup$
– PineDoors
Jan 13 at 4:18












$begingroup$
Which properties are preserved?
$endgroup$
– zoli
Jan 13 at 6:38




$begingroup$
Which properties are preserved?
$endgroup$
– zoli
Jan 13 at 6:38












$begingroup$
Just a few small observations: the orders of the generators are $2,2,4,infty$. Next, the first generator is the square of the third one, so we can throw it out. Next, the second generator acts by inversion on the last two, so the subgroup generated by the last two is normal of index at most $2$, so we can focus on that.
$endgroup$
– verret
Jan 14 at 1:49






$begingroup$
Just a few small observations: the orders of the generators are $2,2,4,infty$. Next, the first generator is the square of the third one, so we can throw it out. Next, the second generator acts by inversion on the last two, so the subgroup generated by the last two is normal of index at most $2$, so we can focus on that.
$endgroup$
– verret
Jan 14 at 1:49












0






active

oldest

votes











Your Answer





StackExchange.ifUsing("editor", function () {
return StackExchange.using("mathjaxEditing", function () {
StackExchange.MarkdownEditor.creationCallbacks.add(function (editor, postfix) {
StackExchange.mathjaxEditing.prepareWmdForMathJax(editor, postfix, [["$", "$"], ["\\(","\\)"]]);
});
});
}, "mathjax-editing");

StackExchange.ready(function() {
var channelOptions = {
tags: "".split(" "),
id: "69"
};
initTagRenderer("".split(" "), "".split(" "), channelOptions);

StackExchange.using("externalEditor", function() {
// Have to fire editor after snippets, if snippets enabled
if (StackExchange.settings.snippets.snippetsEnabled) {
StackExchange.using("snippets", function() {
createEditor();
});
}
else {
createEditor();
}
});

function createEditor() {
StackExchange.prepareEditor({
heartbeatType: 'answer',
autoActivateHeartbeat: false,
convertImagesToLinks: true,
noModals: true,
showLowRepImageUploadWarning: true,
reputationToPostImages: 10,
bindNavPrevention: true,
postfix: "",
imageUploader: {
brandingHtml: "Powered by u003ca class="icon-imgur-white" href="https://imgur.com/"u003eu003c/au003e",
contentPolicyHtml: "User contributions licensed under u003ca href="https://creativecommons.org/licenses/by-sa/3.0/"u003ecc by-sa 3.0 with attribution requiredu003c/au003e u003ca href="https://stackoverflow.com/legal/content-policy"u003e(content policy)u003c/au003e",
allowUrls: true
},
noCode: true, onDemand: true,
discardSelector: ".discard-answer"
,immediatelyShowMarkdownHelp:true
});


}
});














draft saved

draft discarded


















StackExchange.ready(
function () {
StackExchange.openid.initPostLogin('.new-post-login', 'https%3a%2f%2fmath.stackexchange.com%2fquestions%2f3071693%2fidentifying-a-group-involving-transformation-of-4-integers%23new-answer', 'question_page');
}
);

Post as a guest















Required, but never shown

























0






active

oldest

votes








0






active

oldest

votes









active

oldest

votes






active

oldest

votes
















draft saved

draft discarded




















































Thanks for contributing an answer to Mathematics Stack Exchange!


  • Please be sure to answer the question. Provide details and share your research!

But avoid



  • Asking for help, clarification, or responding to other answers.

  • Making statements based on opinion; back them up with references or personal experience.


Use MathJax to format equations. MathJax reference.


To learn more, see our tips on writing great answers.




draft saved


draft discarded














StackExchange.ready(
function () {
StackExchange.openid.initPostLogin('.new-post-login', 'https%3a%2f%2fmath.stackexchange.com%2fquestions%2f3071693%2fidentifying-a-group-involving-transformation-of-4-integers%23new-answer', 'question_page');
}
);

Post as a guest















Required, but never shown





















































Required, but never shown














Required, but never shown












Required, but never shown







Required, but never shown

































Required, but never shown














Required, but never shown












Required, but never shown







Required, but never shown







Popular posts from this blog

Mario Kart Wii

What does “Dominus providebit” mean?

Antonio Litta Visconti Arese